A college professor stops at McDonald's every morning for 10 days to get a number 1 value meal costing $5.39. On the 11th day he orders a number 8 value meal costing $4.38.

Which of the following are true?
Select all that apply.

Select one or more:

1) During the first 10 days the professor's standard deviation was more than 0.

2) During the first 10 days the professor's standard deviation was less than 0.

3) During the first 10 days, the professor's standard deviation was 0.

4) It is impossible to tell anything about the professor's standard deviation for the first 10 days.

5) Considering all 11 days, the professor's standard deviation was lower than the standard deviation of the first 10 days.

6) Considering all 11 days, the professor's standard deviation was higher than the standard deviation of the first 10 days.

7) Considering all 11 days, the professor's standard deviation was the same as the standard deviation of the first 10 days.

8) Considering all 11 days, It is impossible to tell anything about the professor's standard deviation compared to the first 10 days

Answers

Answer 1

The following statements are true:

1. During the first 10 days the professor's standard deviation was more than 0.

4. It is impossible to tell anything about the professor's standard deviation for the first 10 days.

6. Considering all 11 days, the professor's standard deviation was higher than the standard deviation of the first 10 days.

How to explain the information

The standard deviation is a measure of how spread out a set of data is. In this case, the data is the prices of the value meals that the professor orders. If all 10 of the first meals cost $5.39, then the standard deviation would be 0.

This is because there is no variation in the data. However, on the 11th day, the professor orders a meal that costs $4.38. This adds variation to the data, which means that the standard deviation will be greater than 0.

Learn more about standard deviation

https://brainly.com/question/475676

#SPJ4


Related Questions

a) Use the Product Rule to find the derivative of the given function b) Find the derivative by multiplying the expressions first
F(x)=2x^4 (x²-2x)
a) Use the Product Rule to find the derivative of the function. Select the correct answer below and fill in the answer box(es) to comple
A. The derivative is (x²-2x) _____
B. The derivative is 2x^4___+(x²-2) ____
C. The derivative is 2x^4___+24x²____
D. The derivative is 2x^4 (x²-2x)____

Answers

Therefore, the correct option is (C) The derivative is 2x^4___+24x²____.

a) Use the Product Rule to find the derivative of the function. Select the correct answer below and fill in the answer box(es) to complete.

The Product Rule is a method used to take the derivative of a product of functions.

Here is the product rule:(fg)' = f'g + fg'Given function F(x) = 2x^4 (x² - 2x),

Let's differentiate it using the product rule;

f(x) = 2x^4g(x)

= (x² - 2x)f'(x)

= 8x³g'(x)

= 2x - 2

Therefore, (fg)' = f'g + fg'(2x^4) (x² - 2x)'

= f'g + fg'2x^4(2x - 2)

= f'g + fg'(2x^3)(x² - 2x)

= f'g + fg'(2x³)(x² - 2x) = f'g + fg'

Now we will evaluate f'g + fg'f'g = (2x³)(x² - 2x)f'g = 2x^5 - 4x^4fg'

= (2x^4)(2x - 2)fg'

= 4x^5 - 4x^4

Now we substitute the values of f'g and fg'f'g + fg' = (2x³)(x² - 2x) + (2x^4)(2x - 2)f'g + fg' = 2x^5 - 4x^4 + 4x^5 - 4x^4f'g + fg' = 6x^5 - 8x^4

We have thus obtained the derivative.

Hence, the correct option is (B) The derivative is 2x^4___+(x²-2) ____.

b) Find the derivative by multiplying the expressions first

Let's simplify the expression first;

F(x) = 2x^4 (x² - 2x) = 2x^4.x² - 2x.2x^4F(x) = 2x^6 - 4x^5

Now let's differentiate the simplified expression;

F(x) = 2x^6 - 4x^5F'(x) = 12x^5 - 20x^4

To know more about Product Rule visit:

https://brainly.com/question/29198114

#SPJ11

Find the general solution of the following differential equation using the method of undetermined coefficients: d^2y/dx-5 dy/dx +6y=e^3x.

Answers

A = 1/6. So the particular solution is:

y_p = (1/6)e^(3x)

The general solution is then:

y = y_h + y_p = c1e^(2x) + c2e^(3x) + (1/6)e^(3x)

To solve this differential equation using the method of undetermined coefficients, we first find the homogeneous solution by solving the characteristic equation:

r^2 - 5r + 6 = 0

This factors as (r - 2)(r - 3) = 0, so the roots are r = 2 and r = 3. Therefore, the homogeneous solution is:

y_h = c1e^(2x) + c2e^(3x)

Next, we need to find a particular solution for the non-homogeneous term e^(3x). Since this term is an exponential function with the same exponent as one of the roots of the characteristic equation, we try a particular solution of the form:

y_p = Ae^(3x)

Taking the first and second derivatives of y_p gives:

y'_p = 3Ae^(3x)

y"_p = 9Ae^(3x)

Substituting these expressions into the original differential equation yields:

(9Ae^(3x)) - 5(3Ae^(3x)) + 6(Ae^(3x)) = e^(3x)

Simplifying this expression gives:

(9 - 15 + 6)Ae^(3x) = e^(3x)

Therefore, A = 1/6. So the particular solution is:

y_p = (1/6)e^(3x)

The general solution is then:

y = y_h + y_p = c1e^(2x) + c2e^(3x) + (1/6)e^(3x)

where c1 and c2 are constants determined from any initial conditions given.

Learn more about solution from

https://brainly.com/question/27894163

#SPJ11

Rework problem 19 from section 2.3 of your text involving
congressional committees. Assume that the committee consists of 8
Republicans and 4 Democrats. A subcommittee consisting of 5 people
is to be

Answers

The number of possible subcommittees consisting of 5 people from a committee of 8 Republicans and 4 Democrats is 1.

Based on the limited information provided, let's assume that the problem involves selecting a subcommittee consisting of 5 people from a committee consisting of 8 Republicans and 4 Democrats. We need to determine the number of different possible subcommittees that can be formed.

To solve this, we can use the concept of combinations. The number of combinations, denoted as "nCk," represents the number of ways to choose k items from a set of n items without regard to their order.

In this case, we want to calculate 5C5 since we need to select all 5 members for the subcommittee.

Using the formula for combinations, we have:

5C5 = 5! / (5!(5-5)!) = 5! / (5! * 0!) = 5! / 5! = 1

Therefore, there is only one possible subcommittee that can be formed, assuming we select all 5 members.

To learn more about “combinations” refer to the https://brainly.com/question/28065038

#SPJ11

Theory Question DI/HD level Using the standard 22number tutorial with unchanged code, I can see a spinning cube. I then set the following variables as shown below. GLfloat num9_lookAtX =0; GLfloat num10_lookAtY = 0; GLfloat num11_lookAtZ = -200; I ran the program, I can still see the cube. Given I am now looking at a point far past the far plane and nowhere near the cube, why can I still see it?

Answers

The cube is still visible because of depth buffering, which prioritizes the closest objects at each pixel, allowing the cube to be rendered and seen despite being outside the defined frustum.



The reason you can still see the cube despite looking at a point far past the far plane and nowhere near the cube is due to the rendering and projection techniques used in computer graphics. In OpenGL, objects are transformed and projected onto a 2D viewport for display.

The projection matrix, typically defined using functions like gluPerspective or glFrustum, sets the parameters for the clipping planes, including the near and far planes. These planes define the range of depth values that will be rendered. Objects outside this range are clipped and not displayed.However, even though your camera is positioned far beyond the cube and outside the defined frustum, the cube may still be visible due to depth buffering. Depth buffering ensures that only the closest objects at each pixel are displayed. As a result, if the cube is the closest object at certain pixels, it will still be rendered and visible, even though it is technically outside the frustum.

Therefore, The cube is still visible because of depth buffering, which prioritizes the closest objects at each pixel, allowing the cube to be rendered and seen despite being outside the defined frustum.

To learn more about matrix click here

brainly.com/question/29000721

#SPJ11

Verify if the provided y is a solution to the corresponding ODE y=5e^αx
y=e ^2x y′ +y=0
y ′′ −y′ =0

Answers

The result is equal to zero, the provided y = e^(2x) is a solution to the ODE y'' - y' = 0.

To verify if the provided y is a solution to the given ODE, we need to substitute it into the ODE and check if the equation holds true.

y = 5e^(αx)

For the first ODE, y' + y = 0, we have:

y' = d/dx(5e^(αx)) = 5αe^(αx)

Substituting y and y' into the ODE:

y' + y = 5αe^(αx) + 5e^(αx) = 5(α + 1)e^(αx)

Since the result is not equal to zero, the provided y = 5e^(αx) is not a solution to the ODE y' + y = 0.

y = e^(2x)

For the second ODE, y'' - y' = 0, we have:

y' = d/dx(e^(2x)) = 2e^(2x)

y'' = d^2/dx^2(e^(2x)) = 4e^(2x)

Substituting y and y' into the ODE:

y'' - y' = 4e^(2x) - 2e^(2x) = 2e^(2x)

Since the result is equal to zero, the provided y = e^(2x) is a solution to the ODE y'' - y' = 0.

Learn more about  solution from

https://brainly.com/question/27894163

#SPJ11

evaluate the expression, (gof )(x), given the following functions. f(x)=x+2 and g(x)=x^(2)

Answers

The expression (gof)(x) simplifies to [tex]x^2[/tex] + 4x + 4.

To evaluate the expression (gof)(x), we need to substitute the function f(x) into g(x) and simplify the resulting expression.

f(x) = x + 2

g(x) = [tex]x^2[/tex]

Substituting f(x) into g(x), we have:

(gof)(x) = g(f(x))

Replacing f(x) with its value:

(gof)(x) = g(x + 2)

Now, substituting g(x) = [tex]x^2:[/tex]

(gof)(x) = (x + 2)^2

Expanding the square:

(gof)(x) = (x + 2)(x + 2)

(gof)(x) = x^2 + 4x + 4

Therefore, the expression (gof)(x) simplifies to:

(gof)(x) = x^2 + 4x + 4

To know more about expression refer here :

https://brainly.com/question/28170201#

#SPJ11

Suppose that a random sample of 18 adults has a mean score of 64 on a standardized personality test, with a standard deviation of 4. (A higher score indicates a more personable participant.) If we assume that scores on this test are normally distributed, find a 95% confidence interval for the mean score of all takers of this test. Give the lower limit and upper limit of the 95% confidence interval.
Carry your Intermediate computations to at least three decimal places. Round your answers to one decimal place. (If necessary, consult a list of formulas.)
Lower limit:
Upper limit:

Answers

To find the 95% confidence interval for the mean score of all takers of the test, we can use the formula:

Confidence Interval = sample mean ± (critical value * standard error)

First, we need to calculate the critical value. Since the sample size is 18 and we want a 95% confidence level, we look up the critical value for a 95% confidence level and 17 degrees of freedom (n-1) in the t-distribution table. The critical value is approximately 2.110.

Next, we calculate the standard error, which is the standard deviation of the sample divided by the square root of the sample size:

Standard Error = standard deviation / sqrt(sample size)

              = 4 / sqrt(18)

              ≈ 0.943

Now we can calculate the confidence interval:

Confidence Interval = sample mean ± (critical value * standard error)

                   = 64 ± (2.110 * 0.943)

                   ≈ 64 ± 1.988

                   ≈ (62.0, 66.0)

Therefore, the 95% confidence interval for the mean score of all takers of the test is approximately (62.0, 66.0). The lower limit is 62.0 and the upper limit is 66.0.

Learn more about confidence interval here:

https://brainly.com/question/32546207

#SPJ11

Using your calculator matrix mode, solve the system of equations using the inverse of the coefficient matrix. Show all matrices. Keep three decimal places in your inverse matrix. x−2y=−33x+y=2​

Answers

The solution of the given system of equations is [tex]$\left(\begin{matrix}-1 \\ -\frac{17}{7}\end{matrix}\right)$ .[/tex]

Given system of equations: x - 2y = -3x + y = 2We can represent it as a matrix:[tex]$$\left(\begin{matrix}1 & -2 \\ 3 & 1\end{matrix}\right)\left(\begin{matrix}x \\ y\end{matrix}\right) = \left(\begin{matrix}-3 \\ 2\end{matrix}\right)$$[/tex].Let's name this matrix A. Then the system can be written as:[tex]$$A\vec{x} = \vec{b}$$[/tex] We need to find inverse of matrix A:[tex]$$A^{-1} = \frac{1}{\det(A)}\left(\begin{matrix}a_{22} & -a_{12} \\ -a_{21} & a_{11}\end{matrix}\right)$$where $a_{ij}$[/tex]are the elements of matrix A. Let's calculate the determinant of A:[tex]$$\det(A) = \begin{vmatrix}1 & -2 \\ 3 & 1\end{vmatrix} = (1)(1) - (-2)(3) = 7$$[/tex]

Now, let's calculate the inverse of A:[tex]$$A^{-1} = \frac{1}{7}\left(\begin{matrix}1 & 2 \\ -3 & 1\end{matrix}\right)$$[/tex]We can solve the system by multiplying both sides by [tex]$A^{-1}$:$$A^{-1}A\vec{x} = A^{-1}\vec{b}$$$$\vec{x} = A^{-1}\vec{b}$$[/tex]Substituting the values, we get:[tex]$$\vec{x} = \frac{1}{7}\left(\begin{matrix}1 & 2 \\ -3 & 1\end{matrix}\right)\left(\begin{matrix}-3 \\ 2\end{matrix}\right)$$$$\vec{x} = \frac{1}{7}\left(\begin{matrix}-7 \\ -17\end{matrix}\right)$$$$\vec{x} = \left(\begin{matrix}-1 \\ -\frac{17}{7}\end{matrix}\right)$$[/tex]

Let's learn more about matrix:

https://brainly.com/question/31397722

#SPJ11

Factor the following function by finding all rational and other zeros first: P(x)=x^(3)+2x^(2)+x+2.

Answers

The complete factorization of the function P(x) is [tex]P(x) = (x + 1)(x - [-1 + i*\sqrt{ (7)/ 2} (x - [-1 - i*\sqrt{(7)] / 2}.[/tex]

The function given to us is: P(x) = x³ + 2x² + x + 2

To find all the rational and other zeros of the given function, we can use the rational root theorem. According to the rational root theorem, if a polynomial function has a rational zero, then it must be of the form: p/q where p is a factor of the constant term of the function and q is a factor of the leading coefficient of the function.

Here, the constant term is 2 and the leading coefficient is 1, so the possible rational roots of the function P(x) are: ±1, ±2.

Next, we can test these possible rational roots using synthetic division:

Let's start with the root x = -1, we have the following synthetic division:

x  |  1  2  1  2-1  |___|_______|_______|______|1   1   2  |  0

Since we get a zero remainder, x = -1 is a root of the function P(x).Using the factor theorem, we can write:

P(x) = (x + 1)(x² + x + 2)

Now, we need to find the roots of the quadratic factor x² + x + 2. Since there are no real roots of this quadratic, we can use the quadratic formula to find the complex roots:

x = [-b ± sqrt(b² - 4ac)] / 2a

Here, a = 1, b = 1, c = 2, so we have:

[tex]x = [-1 ± sqrt(1 - 4(1)(2))] / 2[/tex]

[tex]= [-1 ± sqrt(-7)] / 2[/tex]

[tex]= [-1 ± i*sqrt(7)] / 2[/tex]

To know more about the function, visit:

https://brainly.com/question/29633660

#SPJ11

HELP PLEASE WILL MARK BRAINLIEST. Leo walk 7km outh then 12km eat. How far i he from the tarting point

Answers

Leo is approximately 13.928 km away from the starting point.

Given that Leo walked 7 km south and then 12 km east, we need to determine the distance from the starting point,

To determine the distance from the starting point, we can use the Pythagorean theorem, which states that in a right triangle, the square of the hypotenuse (the side opposite the right angle) is equal to the sum of the squares of the other two sides.

In this case, the distance Leo walked south forms one side of a right triangle, and the distance he walked east forms the other side. The distance from the starting point will be the length of the hypotenuse.

Using the Pythagorean theorem, we can calculate the distance from the starting point as follows:

Distance² = (7 km)² + (12 km)²

Distance² = 49 km² + 144 km²

Distance² = 193 km²

Taking the square root of both sides gives us:

Distance = √(193)

Distance ≈ 13.928 km

Therefore, Leo is approximately 13.928 km away from the starting point.

Learn more about Pythagorean theorem click;

https://brainly.com/question/14930619

#SPJ4

Complete question =

Leo walk 7km south then 12km east. How far is he from the starting point?

A 27-year-old woman comes to the office due to joint pain. Her symptoms began 10 days ago and consist of bilateral pain in the metacarpophalangeal joints, proximal interphalangeal joints, wrists, knees, and ankles. She describes joint stiffness lasting 10-15 minutes on awakening in the morning. The patient has also had associated fatigue and a few episodes of loose bowel movements associated with mild skin itching and patchy redness. She has no fever, weight loss, or lymphadenopathy. She has no other medical conditions and takes no medications. The patient is married and has 2 children. She works as an elementary school teacher. On examination, there is tenderness of the involved joints without swelling or redness. The remainder of the physical examination is unremarkable. Which of the following is most likely elevated in this patient? A Anti-cyclic citrullinated peptide antibodies B Anti-double-stranded DNA antibodies с Antinuclear antibodies D Anti-parvovirus B19 IgM antibodies E Anti-streptolysin titer F Cryoglobulin levels G Rheumatoid factor

Answers

Antinuclear antibodies (ANAs) are most likely to be elevated in this patient. The correct answer is option C.

In this situation, the patient's most likely diagnosis is lupus erythematosus. Lupus erythematosus is a complex autoimmune disorder that affects the body's normal functioning by damaging tissues and organs. ANA testing is used to help identify individuals who have an autoimmune disorder, such as lupus erythematosus or Sjogren's syndrome, which are two common autoimmune disorders.

Antibodies to specific nuclear antigens, such as double-stranded DNA and anti-cyclic citrullinated peptide (anti-CCP) antibodies, are also found in lupus erythematosus and rheumatoid arthritis, respectively. However, these antibodies are less common in other autoimmune disorders, whereas ANAs are found in a greater number of autoimmune disorders, which makes them a valuable initial screening test.

Learn more about Antinuclear antibodies here:

https://brainly.com/question/31835027

#SPJ11

A United Nations report shows the mean family income for Mexican migrants to the United States is $26,450 per year. A FLOC (Farm Labor Organizing Committee) evaluation of 23 Mexican family units reveals a mean to be $37,190 with a sample standard deviation of $10,700. Does this information disagree with the United Nations report? Apply the 0.01 significance level.

(a) State the null hypothesis and the alternate hypothesis.

H0: µ = ________

H1: µ ? _________

(b) State the decision rule for .01 significance level. (Round your answers to 3 decimal places.)

Reject H0 if t is not between_______ and __________.

(c) Compute the value of the test statistic. (Round your answer to 2 decimal places.)

Value of the test statistic __________

(d) Does this information disagree with the United Nations report? Apply the 0.01 significance level.

Answers

(a) Null hypothesis (H₀): µ = $26,450

Alternate hypothesis (H1): µ ≠ $26,450

Reject H₀ if t is not between -2.807 and 2.807.

(c) Value of the test statistic 3.184.

(d) The information disagrees with the United Nations report at the 0.01 significance level since the calculated t-value falls outside the critical value range.

(a) State the null hypothesis and the alternate hypothesis:

The mean family income for Mexican migrants is $26,450 per year

H₀: µ = $26,450

The mean family income for Mexican migrants is not equal to $26,450 per year.

H₁: µ ≠ $26,450.

(b)

Reject H₀ if t is not between -2.807 and 2.807 (critical values for a two-tailed t-test with 22 degrees of freedom and a significance level of 0.01).

(c) Compute the value of the test statistic:

To compute the test statistic (t-value), we need the sample mean, the hypothesized population mean, the sample standard deviation, and the sample size.

Sample mean (X) = $37,190

Hypothesized population mean (µ) = $26,450

Sample standard deviation (s) = $10,700

Sample size (n) = 23

t-value = (X - µ) / (s / √n)

= ($37,190 - $26,450) / ($10,700 / √23)

= ($37,190 - $26,450) / ($10,700 / √23)

= $10,740 / ($10,700 / √23)

= 3.184

The calculated t-value is approximately 3.184.

d.  To determine if this information disagrees with the United Nations report, we compare the calculated t-value with the critical values for a two-tailed t-test with 22 degrees of freedom and a significance level of 0.01.

The critical values for a two-tailed t-test with a significance level of 0.01 and 22 degrees of freedom are approximately -2.807 and 2.807.

Since the calculated t-value of 3.184 falls outside the range -2.807 to 2.807, we reject the null hypothesis (H0) and conclude that there is evidence to suggest a disagreement with the United Nations report.

Therefore, based on the provided data and significance level, the information disagrees with the United Nations report.

To learn more on Statistics click:

https://brainly.com/question/30218856

#SPJ4

in a firm with a multidivisional structure, the object is to try to achieve tight coordination between functions with emphasis on r

Answers

The statement that In a firm with a multidivisional structure, the object is to try to achieve tight coordination between functions with emphasis on R&D, production, and marketing is false.

What is multidivisional structure?

In this kind of structure, employees are divided into departments based on the types of products and/or geographic areas. For instance, General Electric has six product divisions: energy, capital, home & business solutions, healthcare, aviation, and transportation.

In contrast to a functional organization, which allows for greater efficiency by having only one department oversee all activities in a certain area, such as marketing, a multidivisional structure requires that a corporation have marketing units within each of its divisions.

It is untrue to say that the goal of a company with a multidivisional structure is to create close coordination between functions, with a focus on R&D, manufacturing, and marketing.

Learn more about multidivisional structure   at:

https://brainly.com/question/14992166

#SPJ4

complete question;

In a firm with a multidivisional structure, the object is to try to achieve tight coordination between functions with emphasis on R&D, production, and marketing. TRUE /FALSE

Audric drove 120km from Quezon City to San Pablo, Laguna to attend their family reunion. His average speed for the trip to San Pablo, Laguna was 10k(m)/(h) faster than on the way back to Quezon City, and as a result, his return trip took an hour

Answers

Audric's average speed for the entire trip is 125 km/h.

The speed of Audric during his trip to San Pablo, Laguna from Quezon City is 10 km/h faster than his speed on his way back to Quezon City. His return trip took an hour.

Find Audric's average speed for the entire trip.

Audric drove 120 km from Quezon City to San Pablo, Laguna to attend their family reunion.

Let's assume the speed of Audric on his way to San Pablo, Laguna was x km/h.

So, his speed on his way back to Quezon City was (x - 10) km/h.

Using the formula:

speed = distance/time

We can calculate the time Audric took to reach San Pablo, Laguna and his time to return to Quezon City.

Audric's time to reach San Pablo, Laguna = 120/xAudric's time to return to Quezon City

= 120/(x - 10)

According to the problem, his return trip took an hour,

so we have:

120/(x - 10) = 1

Now we can solve for x as follows:

120 = x - 10120 + 10

= xx = 130 km/h

Therefore, Audric's speed on his way to San Pablo, Laguna was 130 km/h, and his speed on his way back to Quezon City was (130 - 10) = 120 km/h.

Now, we can find Audric's average speed for the entire trip as follows:

Average speed = total distance / total time

Total distance = 120 km + 120 km = 240 km

Total time = 120/130 + 120/120

= 0.92 + 1 hours

= 1.92 hours

Average speed = 240/1.92

= 125 km/h

To know more about speed visit :

brainly.com/question/31756299

#SPJ11

Find (f∘g)(x) for the indicated functions and simplify. f(x)=8x+2,g(x)=7/(x+2)​

Answers

The composition (f∘g)(x) is given by (2x + 60)/(x + 2).

To find (f∘g)(x), we need to substitute the function g(x) into the function f(x) and simplify the expression.

f(x) = 8x + 2

g(x) = 7/(x + 2)

To find (f∘g)(x), we substitute g(x) into f(x):

(f∘g)(x) = f(g(x))

Substituting g(x) into f(x):

(f∘g)(x) = 8(g(x)) + 2

Replacing g(x) with its value:

(f∘g)(x) = 8(7/(x + 2)) + 2

Now, let's simplify the expression:

(f∘g)(x) = (56/(x + 2)) + 2

To simplify further, we need to obtain a common denominator:

(f∘g)(x) = (56/(x + 2)) + (2(x + 2)/(x + 2))

Combining the fractions:

(f∘g)(x) = (56 + 2(x + 2))/(x + 2)

Simplifying the numerator:

(f∘g)(x) = (56 + 2x + 4)/(x + 2)

(f∘g)(x) = (2x + 60)/(x + 2)

Therefore, the simplified expression for (f∘g)(x) is (2x + 60)/(x + 2).

To learn more about functions visit : https://brainly.com/question/11624077

#SPJ11

A projectile is thrown upward so that its distance above the ground, in feet, after t seconds is h=-11t^(2)+374t. After how many seconds does it reach its maximum height

Answers

The projectile reaches its maximum height after 17 seconds.

To find the time at which the projectile reaches its maximum height, we need to determine the vertex of the parabolic function h(t) = -11t^2 + 374t. The vertex represents the maximum point on the graph of the function.

The equation of the vertex can be found using the formula t = -b / (2a), where a and b are the coefficients of the quadratic equation in standard form (at^2 + bt + c).

In our case, the equation is h(t) = -11t^2 + 374t, so a = -11 and b = 374. Plugging these values into the formula, we get:

t = -374 / (2 * -11)

t = -374 / -22

t = 17

Therefore, the value of maximum height obtained is 17.

To know more about quadratic equation refer here:

https://brainly.com/question/29269455#

#SPJ11

in the quadratic equation the square of the sum of two consecutive even numbers is 324. what are the integers

Answers

Let x and x + 2 be two consecutive even numbers. Then, according to the problem, we can form a quadratic equation that represents the sum of the square of two consecutive even numbers.

This quadratic equation is shown as follows: [tex](x + x + 2)² = 324[/tex]Simplify the left-hand side of the equation as shown below. (2x + 2)² = 324Expand the left-hand side of the equation as shown below.

[tex]2² × (x² + 2x + 1) = 324[/tex]

Simplify the equation as shown below.

[tex]4(x² + 2x + 1) = 324[/tex]Simplify the equation as shown below.4x² + 8x - 320 = 0Divide the whole equation by 4 as shown below .[tex]x² + 2x - 80 = 0[/tex]Factor the quadratic expression as shown below.[tex](x + 10)(x - 8) = 0[/tex] Therefore, the two integers that satisfy the given quadratic equation are 10 and -8.

To know more about equation visit:

https://brainly.com/question/29657983

#SPJ11

If the correlation between amount of heating oil in gallons and housing price is - 0.86, then which one is the best one to describe the relationship between two variables?
a.Amount of heating oil in gallons and housing price are weakly negatively linearly related.
b.Amount of heating oil in gallons and housing price are weakly negatively related.
c.Amount of heating oil in gallons and housing price are highly negatively related.
d.Amount of heating oil in gallons and housing price are highly negatively linearly related.

Answers

d. Amount of heating oil in gallons and housing price are highly negatively linearly related.

The correlation coefficient (-0.86) indicates a strong negative linear relationship between the amount of heating oil in gallons and housing price. The closer the correlation coefficient is to -1 or 1, the stronger the linear relationship. In this case, the correlation coefficient of -0.86 suggests a strong negative linear relationship between the two variables.

To know more about linear visit:

brainly.com/question/31510530

#SPJ11

A company rents moving trucks out of two locations: St. Louis and Tampa. Some of their customers rent a truck in one city and return it in the other city, and the rest of their customers rent and return the truck in the same city. The company owns a total of 400 trucks. The company has seen the following trend: • About 30 percent of the trucks in St. Louis move to Tampa each week. • About 60 percent of the trucks in Tampa move to St. Louis each week. Suppose right now St. Louis has 330 trucks. How many trucks will be in each city after 1 week? [Round answers to the nearest whole number.] St. Louis: Tampa: If the vector i represents the distribution of trucks, where I1 is the number in St. Louis and 12 is the number in Tampa, find the matrix A so that Až is the distribution of trucks after 1 week. A = How many trucks will be in each city after 4 weeks? [Round answers to the nearest whole number.] St. Louis: Tampa: A brass manufacturer makes three different type of wholesale brass blocks from copper and zinc acco to the following matrix. Brass Blends Muntz metal 60 % 40 % High brass 65 % 35 % Copper Zinc Gilding metal 95 % 5% a) Make a 2 x 3 matrix B that contains the blending information in decimal form. In addition, the demand (in thousands of pounds) from Plant 1 is 10 High Brass, 3 Muntz metal, and 27 Gilding metal, and the demand from Plant 2 is is 12 High Brass, 3 Muntz metal, and 28 Gilding metal. b) Make a 3 x 2 matrix D for the demands at each plant. C) Find the matrix product to find each locations need for each type of metal. d) if the price of zinc is 50.58 per pound and the price of copper is 53.35 per pound. The total cost of Plant 1 is The total cost of plant 2 is

Answers

1. After 1 week, truck in St. Louis is 221 and in Tampa is 348.

a)  Blending matrix B: [tex]\left[\begin{array}{ccc}0.35&0.65&0\\0.4&0.6&0\\0.05&0.95&0\end{array}\right][/tex]  

b) Demand matrix D:  [tex]\left[\begin{array}{ccc}10&3&27\\12&3&28\end{array}\right][/tex]  

c) C = [tex]\left[\begin{array}{ccc}6.05&33.95&0\\6.8&36.2&0\end{array}\right][/tex]

d) The total cost of Plant 1 is $51.69 and the total cost of Plant 2 is $51.58.

Given information:

St. Louis currently has 330 trucks.About 30% of the trucks in St. Louis move to Tampa each week.About 60% of the trucks in Tampa move to St. Louis each week.

1. We can represent the distribution of trucks using a vector. Let the number of trucks in St. Louis as I1 and the number of trucks in Tampa as I2.

The change in the number of trucks in St. Louis is

= -0.3 x 330

= -99.

and, the change in the number of trucks in Tampa is

= 0.6 (400 - 330)

= 18.

Therefore, after 1 week, the number of trucks in St. Louis

= 330 - 99

= 231,

and the number of trucks in Tampa

= 330 + 18

= 348

a) Blending matrix B:

                                B = [tex]\left[\begin{array}{ccc}0.35&0.65&0\\0.4&0.6&0\\0.05&0.95&0\end{array}\right][/tex]  

b) Demand matrix D:

                              D = [tex]\left[\begin{array}{ccc}10&3&27\\12&3&28\end{array}\right][/tex]  

c) Matrix product:

To calculate the locations' needs for each type of metal, we can multiply matrix D by matrix B:

C = D x B

                    C =    [tex]\left[\begin{array}{ccc}10&3&27\\12&3&28\end{array}\right][/tex]  [tex]\left[\begin{array}{ccc}0.35&0.65&0\\0.4&0.6&0\\0.05&0.95&0\end{array}\right][/tex]  

                     C = [tex]\left[\begin{array}{ccc}6.05&33.95&0\\6.8&36.2&0\end{array}\right][/tex]

d) Total cost of Plant 1 = sum(C[0] x [50.58, 53.35])

Total cost of Plant 2 = sum(C[1] x [50.58, 53.35])

Performing the calculations will give us the total costs.

Total cost of Plant 1 = $51.69

and, Total cost of Plant 2 = (0.65 x $50.58) + (0.35 x $53.35)

                                          = $32.90 + $18.68

                                          = $51.58

Therefore, the total cost of Plant 1 is $51.69 and the total cost of Plant 2 is $51.58.

Learn more about Matrix here:

https://brainly.com/question/29132693

#SPJ4

2 What is the slope -intercept form of the equation passing through the point (2,-1) and parallel to the line y=-4.3x+6.7?

Answers

Therefore, the slope-intercept form of the equation passing through the point (2, -1) and parallel to the line y = -4.3x + 6.7 is y = -4.3x + 7.6.

To find the slope-intercept form of the equation passing through the point (2, -1) and parallel to the line y = -4.3x + 6.7, we need to determine the slope of the parallel line first. The slope of the given line is -4.3 since it is in the form y = mx + b, where m represents the slope. Since the line we are looking for is parallel to this line, it will have the same slope. Now, we can use the point-slope form of a linear equation to find the equation of the line. The point-slope form is given by y - y1 = m(x - x1), where (x1, y1) is the given point and m is the slope.

Substituting the values, we have:

y - (-1) = -4.3(x - 2)

Simplifying the equation:

y + 1 = -4.3x + 8.6

Next, we can convert this equation to the slope-intercept form, y = mx + b, by isolating y:

y = -4.3x + 8.6 - 1

y = -4.3x + 7.6

To know more about equation,

https://brainly.com/question/19229281

#SPJ11

the population of the town of chestnut hulls increased at a steady rate from 19,800 in 2001 to 21,400 in 2010. on average which towns population grew faster? what was the average rate of growth for the fastest growing town?

Answers

Chestnut Hills grew faster with an average growth rate of 1,600 people per decade, while the growth rate for Walnut Park is unknown based on the given information.

The correct answer is option D.

Based on the information provided, we can calculate the average rate of growth for each town and determine which town grew faster.

For Chestnut Hills:

Population in 2001 = 19,800

Population in 2010 = 21,400

Number of years = 2010 - 2001 = 9

Change in population = 21,400 - 19,800 = 1,600

Average rate of growth = Change in population / Number of years = 1,600 / 9 = 177.78 (rounded to the nearest whole number)

For Walnut Park, the graph does not provide specific population values for each year, so we cannot calculate the exact rate of growth. However, based on the given options, we can conclude that the average rate of growth for Walnut Park must be less than 1,600 people per decade, as Chestnut Hills had a growth rate of 1,600 people per decade.

Therefore, the correct answer is: D. Chestnut Hills grew faster. It grew by 1,600 people per decade.

For more such information on: growth rate

https://brainly.com/question/30611694

#SPJ8

A $2,800 loon is Paid bock with simple interest. If the omount Poid beck wo $3,388, Whot Was the simple interest?

Answers

the simple interest is $588.

To find the simple interest, we need to subtract the principal amount (initial loan) from the total amount paid back.

Simple Interest = Total Amount Paid Back - Principal Amount

In this case:

Principal Amount = $2,800

Total Amount Paid Back = $3,388

Simple Interest = $3,388 - $2,800

Simple Interest = $588

Therefore, the simple interest is $588.

Learn more about simple interest:

https://brainly.com/question/25845758

#SPJ11

Calculate the cross product assuming that u×w=⟨5,−6,−1⟩ (4u+4w)×w=

Answers

The cross product assuming that is  (4u + 4w) × w = 4(u × w) + 0(4u + 4w) × w = 4(u × w) = 4⟨5, −6, −1⟩= ⟨20, −24, −4⟩

Given that u × w = ⟨5, −6, −1⟩

We are to find (4u + 4w) × w

We know that(4u + 4w) × w = 4(u + w) × w ......(i)u × w = |u| |w| sin θwhere, |u| = magnitude of vector

uw = angle between u and w

As we can see, we are not given the magnitude of either u or w, and nor are we given the angle between them.

Hence, we cannot calculate the vector product using the above formula.

However, we can use the following identity which will give us a useful result:

                                         (u + v) × w = u × w + v × w

So, we can write(4u + 4w) × w = (4u × w) + (4w × w)

Expanding, we get(4u + 4w) × w = 4(u × w) + 0(4u + 4w) × w = 4(u × w) = 4⟨5, −6, −1⟩= ⟨20, −24, −4⟩

Thus, the detailed answer is  (4u + 4w) × w = 4(u × w) + 0(4u + 4w) × w = 4(u × w) = 4⟨5, −6, −1⟩= ⟨20, −24, −4⟩

Learn more about cross product

brainly.com/question/29097076

#SPJ11

Dr. Wahl made a cup of coffee and she likes to drink it as soon as it cools to 130 ∘ F. The coffee is 194 ^0 F when she places the coffee on the counter to cool and after one minute, the coffee is 168 ∘ F. If the ambient temperature is a constant 70 ∘ F, how long until she drinks the coffee? The Mathematics behind the Model: In this problem we use Newton's Law of Cooling which states that the rate at which the temperature of the coffee is changing is proportional to the difference between the ambient temperature and the temperature of the coffee. The idea of proportionality is a common one in mathematics so we can review it here: a quantity z is (directly) proportional to x if z=kx, for some constant k. Let's fix a common notation for our work. Let T(t) be the temperature in degrees Fahrenheight of the coffee at time t in minutes. Let A represent the ambient air temperature and k be the constant of proportionality. 1. Write the differential equation that expresses Newton's Law of Cooling: the rate at which the temperature of the coffee is changing is proportional to the difference between the ambient temperature (A) and the temperature of the coffee (T). Hint: Use the symbols dT/dt,k,T and A. 2. What is the dependent variable in this differential equation? 3. What is the independent variable in this differential equation? 4. What is the value of the constant A ? Replace A in your differential equation and write the differential equations here. 5. This equation is separable. To solve it, separate (the variables T and t ) and integrate! Without using logarithms, write the function T(t) that solves the differential equation here. Your answer will include constants C _1 =e ^C (where C is the constant of integration) and k. 6. What are the values of T(0) and T(1) ? 7. Use the data points T(0) and T(1) to determine the two constants and write T(t) again here. 8. How much total time passes until she should begin drinking the coffee? Please answer in minutes and seconds to the nearest second.

Answers

The differential equation that expresses Newton's Law of Cooling is:

dT/dt = -k(T - A)

The dependent variable in this differential equation is T, representing the temperature of the coffee.

The independent variable in this differential equation is t, representing time in minutes.

The value of the constant A is 70 ∘ F.

The differential equation is:

dT/dt = -k(T - 70)

To solve the differential equation, we can separate the variables T and t and integrate both sides with respect to t:

1/(T - 70) dT = -k dt

Integrating both sides, we get:

ln|T - 70| = -kt + C_1

where C_1 is a constant of integration.

Exponentiating both sides, we get:

|T - 70| = e^C_1 * e^(-kt)

Now, since T cannot be negative, we can drop the absolute value signs, and we get:

T(t) = Ce^(-kt) + 70

where C = ±e^C_1 is another constant of integration.

We are given T(0) = 194 ∘ F and T(1) = 168 ∘ F. Plugging these values into the equation T(t) = Ce^(-kt) + 70, we get the following two equations:

194 = Ce^0 + 70   -->   C = 124

168 = 124e^(-k) + 70

Solving the second equation for k, we get:

k = ln(54/124)

Plugging in the values of C and k, we get:

T(t) = 124e^(-ln(54/124)t) + 70

Simplifying, we get:

T(t) = 54e^(-0.693t) + 70

We want to find the time it takes for the coffee to cool to 130 ∘ F. This means we need to solve for t in the equation T(t) = 130. Plugging in the values we found for C and k, we get:

130 = 54e^(-0.693t) + 70

Subtracting 70 from both sides and dividing by 54, we get:

0.3704 = e^(-0.693t)

Taking the natural logarithm of both sides, we get:

ln(0.3704) = -0.693t

Solving for t, we get:

t = ln(0.3704)/(-0.693) ≈ 1.93 minutes

Therefore, approximately 1 minute and 56 seconds pass until Dr. Wahl should begin drinking the coffee.

learn more about differential equation here

https://brainly.com/question/32645495

#SPJ11

The length of a niww rectangulat playing field is 8 yardn longer than triple the width It the perimeter of the rectanguiar playing finld is 376 yards. what are its dimensiotis? The wieh is yards

Answers

The rectangular playing field's dimensions are 85 yards by 26 yards, with a width of 26 yards.


Let x be the width of the rectangular playing field. According to the question, the length of a new rectangular playing field is 8 yards longer than triple the width. Therefore, the length of the rectangular playing field will be (3x + 8) yards.

The perimeter of the rectangular playing field is 376 yards. Thus, the formula for the perimeter of a rectangle is P = 2L + 2W, where P is the perimeter, L is the length, and W is the width. Substituting the values of L and W, we get:

2(3x + 8) + 2x = 376

6x + 16 + 2x = 376

8x + 16 = 376

8x = 360

x = 45

Therefore, the width of the rectangular playing field is 45 yards. And the length will be (3(45) + 8) = 143 yards. Hence, the dimensions of the rectangular playing field are 85 yards by 26 yards, with a width of 26 yards.

To know more about rectangle refer here:

https://brainly.com/question/30688709?referrer=searchResults

#SPJ11

Find the volume of the solid formed by rotating the region enclosed by y = e²+4, y = 0, x = 0, and x = 0.5 about the y-axis.

Answers

The volume of the solid formed by rotating the region enclosed by y = e²+4, y = 0, x = 0, and x = 0.5 about the y-axis is approximately 1.28 cubic units.

To calculate the volume, we can use the method of cylindrical shells. The height of each shell is given by the difference between the upper and lower bounds of y, which is e²+4 - 0 = e²+4. The circumference of each shell is given by 2πy, and the thickness of each shell is dx.

Integrating the volume formula

V = ∫(2πy)(dx) over the interval [0, 0.5],

we get ,

V = ∫[0,0.5] (2π(e²+4)) dx = 2π(e²+4)∫[0,0.5] dx = 2π(e²+4)(x)|[0,0.5] = 2π(e²+4)(0.5) = π(e²+4) ≈ 1.28 cubic units.

In summary, the volume of the solid formed by rotating the given region about the y-axis is approximately 1.28 cubic units. The calculation involves integrating the formula for the volume of cylindrical shells, considering the height, circumference, and thickness of each shell.

Learn more about volume here:
brainly.com/question/28058531

#SPJ11

i) Are the following equalities generally valid? A ∪ (B \ C) = (A ∪ B) \ (A ∪ C)
A ∩ (B \ C) = (A ∩ B) \ (A ∩ C)
Give a counterexample or prove the argument
ii) Give an example of a set A containing at least one element that fulfills the condition
if x ∈ A so {x} ∈ A

Answers

1.  The equalities are not generally valid.

2. 0 is an element of A, and {0} is also an element of A since it is a singleton set containing 0.

i) The equalities A ∪ (B \ C) = (A ∪ B) \ (A ∪ C) and A ∩ (B \ C) = (A ∩ B) \ (A ∩ C) are not generally valid.

Counterexample for A ∪ (B \ C) = (A ∪ B) \ (A ∪ C):

Let A = {1, 2}, B = {2, 3}, and C = {1, 3}.

A ∪ (B \ C) = {1, 2} ∪ {2} = {1, 2}

(A ∪ B) \ (A ∪ C) = ({1, 2} ∪ {2, 3}) \ ({1, 2} ∪ {1, 3}) = {1, 2, 3} \ {1, 2} = {3}

Since {1, 2} is not equal to {3}, the equality A ∪ (B \ C) = (A ∪ B) \ (A ∪ C) does not hold in this case.

Counterexample for A ∩ (B \ C) = (A ∩ B) \ (A ∩ C):

Let A = {1, 2}, B = {2, 3}, and C = {1, 3}.

A ∩ (B \ C) = {1, 2} ∩ {2} = {2}

(A ∩ B) \ (A ∩ C) = ({1, 2} ∩ {2, 3}) \ ({1, 2} ∩ {1, 3}) = {2} \ {1, 2} = {}

Since {2} is not equal to {}, the equality A ∩ (B \ C) = (A ∩ B) \ (A ∩ C) does not hold in this case.

Therefore, the equalities are not generally valid.

ii) An example of a set A containing at least one element that fulfills the condition if x ∈ A, then {x} ∈ A is:

A = {0, {0}}

In this case, 0 is an element of A, and {0} is also an element of A since it is a singleton set containing 0.

Learn more about Elements from

https://brainly.com/question/25916838

#SPJ11

Amber's Video Rentals wants to increase the quantity of videos that it sells by 1 percent. The price elasticity of demand for videos sold by Amber's Video Rentals is 0.2. What is the percentage price

Answers

The percentage price increase needed to achieve a 1 percent increase in quantity sold, given a price elasticity of demand of 0.2, would be 0.5 percent.

Price elasticity of demand measures the responsiveness of quantity demanded to a change in price. In this case, the price elasticity of demand is given as 0.2.

The formula for price elasticity of demand is:

Elasticity = (% change in quantity demanded) / (% change in price)

We want to find the percentage price increase needed to achieve a 1 percent increase in quantity sold. Let's denote the percentage change in quantity demanded as 1 percent and the percentage change in price as X percent.

0.2 = (1%)/(X%)

Cross-multiplying and solving for X, we get:

X% = (1%)/0.2

X% = 5%

Therefore, a 5 percent increase in price would result in a 1 percent increase in quantity sold.

To increase the quantity of videos sold by 1 percent, Amber's Video Rentals would need to increase the price by approximately 0.5 percent, given a price elasticity of demand of 0.2.

To know more about percentage follow the link:

https://brainly.com/question/29520143

#SPJ11

Let f (0) = 4 sin(0) sec² (0) + sec(0) tan(0). Find the anti derivative function, F (8), if F (0) = 0.

Answers

The antiderivative function F(x) is given by: F(x) = -4cos(x) - 4/3cot(x) + sec(x) + 4To find the antiderivative function F(x) given that f(0) = 4sin(0)sec^2(0) + sec(0)tan(0) and F(0) = 0, we need to integrate f(x) with respect to x.

First, let's simplify f(x) using trigonometric identities:

f(x) = 4sin(x)sec^2(x) + sec(x)tan(x)

Since sec^2(x) = 1 + tan^2(x), we can rewrite f(x) as:

f(x) = 4sin(x)(1 + tan^2(x)) + sec(x)tan(x)

    = 4sin(x) + 4sin(x)tan^2(x) + sec(x)tan(x)

Now, let's find the antiderivative of f(x) using integration techniques:

∫ f(x) dx = ∫ (4sin(x) + 4sin(x)tan^2(x) + sec(x)tan(x)) dx

We can integrate each term separately:

∫ 4sin(x) dx = -4cos(x) + C1, where C1 is the constant of integration

∫ 4sin(x)tan^2(x) dx = -4/3cot(x) + C2, where C2 is the constant of integration

∫ sec(x)tan(x) dx = sec(x) + C3, where C3 is the constant of integration

Now, we can combine these results to find the antiderivative function F(x):

F(x) = -4cos(x) - 4/3cot(x) + sec(x) + C, where C = C1 + C2 + C3 is the constant of integration

Given that F(0) = 0, we can substitute x = 0 into the expression for F(x):

F(0) = -4cos(0) - 4/3cot(0) + sec(0) + C = -4 + C = 0

From this, we find that C = 4.

Therefore, the antiderivative function F(x) is given by:

F(x) = -4cos(x) - 4/3cot(x) + sec(x) + 4

To learn more about derivative click here:

brainly.com/question/28984102?

#SPJ11

I. Both paira of opposite sides are congruent II. Any two consecutive angles are supplementary III. Both pairs of opposite aides are parallel IV. Both pairs of opposite angles are confruent

Answers

The set of conditions that describe a parallelogram are as follows:

Both pairs of opposite sides are congruent.

Both pairs of opposite sides are parallel.

Both pairs of opposite angles are congruent.

Any two consecutive angles are supplementary.

The above-described properties are used to define a parallelogram. A parallelogram is defined as a quadrilateral with both pairs of opposite sides parallel and congruent.The opposite sides of a parallelogram are congruent. All sides of the parallelogram are parallel to each other. The opposite angles are congruent and are equal in size. The consecutive angles are supplementary, meaning they add up to 180 degrees. The diagonal of a parallelogram bisects each other.

Parallelograms come in a variety of shapes and sizes. The properties of a parallelogram will remain the same regardless of its size or shape. All parallelograms are quadrilaterals, and they are categorized as such because they have four sides.

The parallelogram has several intriguing properties. Its properties, including its opposite sides being parallel and congruent, make it unique.The sum of the interior angles of a parallelogram is always 360 degrees. Additionally, the area of the parallelogram is equivalent to the product of its base and height.


To know more about parallelogram click here:

https://brainly.com/question/32441125

#SPJ11

Other Questions
Hi i need help writing a c program to make TI-RSLK MAX MSP432 to blinkRed -> (1s) -> Green -> (1s) -> Blue -> (1s) -> Red and keep repeating nonstopled color should change every 1 second in order above and should never be turned off the parent of a 24-month-old child asks the primary care pediatric nurse practitioner when toilet training should begin. how will the pediatric nurse practitioner respond? Which of the following are nucleophiles and which are electrophiles? -{OH}, \quad {H}_{2} {O}, {NH}_{3}, \quad {Br} -{OH}_{1} {H} Identify a MIS process or any form of technology in an organisation of your choice that can be improved and or updated. 2 Investigate the problem through discussions with relevant stakeholders. 3. Formulate a problem statement. The problem statement must be concise and unambiguous. Use supporting literature to support your problem statement. 4. Construct a mini literature review through consulting reputable journal articles, text books, reports and other relevant sources. Refrain from referring to wiki sites, organizational websites and blogs. 5. Investigate the best possible solution based on your research by collecting relevant information from the organisation you plan to research. Provide an implementation plan 6. Make recommendations that will assist the organisation to improve its technology, MIS and competitive stance. 7. Provide a conclusion to your report 8. All citations must be included in the reference list at the end of the report True or False. In mla format, if you are using a quotation that is longer than two lines (when you type it in your paper), indent the entire quotation and remove the quotation marks. afb, inc. had earnings per share of $4 per share last year and paid a dividend of $1 per share. for the current year, afb, inc. generated earnings per share of $6 and paid a dividend of $1 per share. this is an example of what type of dividend policy? group of answer choices small, regular dividend plus a year-end extra payout ratio equal to zero stable dollar dividend per share constant dividend payout ratio recording tax valuation allowance maui resort inc. determined that the balance in its deferred tax asset account on december 31, 2020, was $100,000. management reviewed all available positive and negative evidence to estimate that 30% of the deferred tax asset was more likely than not to be realized. the valuation allowance for deferred tax assets has a december 31, 2020, unadjusted balance of $8,000 (credit). record the entry to adjust the allowance on december 31, 2020. Not all managers are leaders and not all leaders are managers. Critically evaluate this statement in light of what you have learnt about management and leadership in this module to date. In your critical evaluation, be sure to reflect on the importance of managers and management in the modern society. Picking and Preparing the Logs For picking and preparing the logs, you have to consider the best tree species, a log calculation, the felling of the trees, debarking, and drying of the logs. Let's assume you will require 100 logs. This requires the felling of 100 trees!l Let's assume one person can fall 20 trees in one day, and can debark and dry 40 logs in one day. Write a formula for a linear function f(x) that models the situation, where x is the number of years after 2007 . In 2007 the average adult ate 54 pounds of chicken. This amount will increase by 0.6 p The depicted cells (each labeled with a letter below) were taken from an organism that has two types of chromosomes.What is the ploidy of Cell C? What service converts natural language names to IP addresses? !DNSHTMLFTPHTTPIP What are the status factors contributing to DigitalTransformation Strategy in BankingSector 1. What do you understand by context? Why is context important? To develop a navigational system for a car, what types of context information will be necessary?2. What is a content-aware system? What can be the types of information needed for developing a fully context-aware system? A small town has 5000 adult males and 3000 adult females. A sociologist conducted a survey and found that 30% of the males and 20% of the females drink heavily. An adult is selected at random from the town. (Enter your probabilities as fractions.)(a) What is the probability the person is a male? (b) What is the probability the person drinks heavily?c) What is the probability the person is a male or drinks heavily? (d) What is the probability the person is a male, if it is known that the person drinks heavily? Suppose that X 1and X 2are independent Unif(1,2,3,4,5,6) random variables. Let X=min {X 1,X 2},Y=max{X 1,X 2}. Answer the following questions: 4.1 (15 points) Calculate P(X=xY=y) Answer 4.2 (15 points) Calculate E[XY=y] nand then verify that E[X]=E[E[XY]] All the snow plowing companies in a market are considering adopting a new process that doesn't require any new inputs, but enables quieter snow removal on roads. Which of the following may prevent the companies in that industry from charging premium prices? Group of answer choices:-Lack of economies of scale-Bargaining power of suppliers-Threat of potential entrants Continue to implement class my_str below. This class defines a sequence of characters similar to the string type. Use a dynamically allocated c-string array of characters so it can be resized as needed. Do not add methods to the class my_str, however feel free to add helper functions. class StringType \{ public : // one parameter constructor constructs this object from a // parameter, s, defaults to the empty string "" // write and use strdup() to implement this constructor, // it allocates a new array, then uses strcpy() to copy // chars from array s to the new array StringType (const char s=" ) \{ // you fill in \} // copy constructor for a StringType, must make a deep copy // of s for this. (you can use strdup() you wrote) StringType( const StringType \& s ) \{ // you fill in \} // move constructor StringType( StringType \&\& s ) noexcept \{ // you fill in \} // assigns this StringType from StringType s (perform deep assig // remember, both this and s have been previously constructed 1 // so they each have storage pointed to by buf StringType\& operator =( const StringType \& s){ // you fill in \} // move assignment operator overload StringType\& operator =( StringType \&\& s ) noexcept \{ // you fill in \} // return a reference to the char at position index, 0 is // the first element and so on // index must be in bounds char\& operator [] (const int index) \{ // you fill in \} int length() const \{ // you fill in \} // returns the index of the first occurance of c in this StringType // indices range from 0 to length()-1 // returns 1 if the character c is not in this StringType int indexOf( char c ) const \{ // you fill in \} // returns the index of the first occurrence of pat in this StringTyp // indices range from 0 to length()-1 // returns 1 if the character string pat is not in this StringType. // write and use strstr() to implement this function int indexOf ( const StringType \& pat ) const \{ // you fill in \} 5 // true if both StringType objects contain the same chars // in same position .. e.g, "abc"=="abc" returns true // write and use strcmp() to implement this function bool operator ==( const StringType \& s ) const \{ // you fill in \} // concatenates this and s to make a new StringType // e.g., "abc"+"def" returns "abcdef" // write and use str2dup() to implement this function, // it should allocate a new array then call strcpy() // and strcat() StringType operator+( const StringType \& s ) const \{ // you fill in \} // concatenates s onto end of this // e.g., s="abc"s+= "def" now s is "abcdef" // use str2dup() StringType\& operator +=( const StringType \& s){ // you fill in \} // returns another StringType that is the reverse of this StringType // e.g., s="abc"; s. reverse() returns "cba" // write strrev(char *dest, char *src) like strcpy() but // copies the reverse of src into dest, then use it StringType reverse() const \{ // you fill in \} // prints out this StringType to the ostream out void print( ostream \& out ) const \{ // you fill in 3 // reads a word from the istream in and this StringType // becomes the same as the characters in that word // use getline() to implement read() void read( istream \& in ) \{ // you fill in \} // destruct a StringType, must free up each node in the head list StringType() \{ // you fill in \} private: char* buffer ; int capacity; // or better: size_t capacity \} // these two I/O methods are complete as long as you define // print and read methods correctly inline ostream\& operator At time t = 0, a vessel contains a mixture of 14 kg of water and an unknown mass of ice in equilibrium at 0C. The temperature of the mixture is measured over a period of an hour, with the following results: During the first 45 min, the mixture remains at 0C; from 45 min to 60 min, the temperature increases steadily from 0C to 2.0C. Neglecting the heat capacity of the vessel, determine the mass of ice that was initially placed in the vessel. Assume a constant power input to the container. Answer is in kg. 3. The cost of preferred stock Blie Panda has preferred stock that pays a dividend of $6,00 per share and sells for $100 per share, It is considering issuing new shares of preferred stock. These new shares incur an underwriting (or flotation) cost of 2.10%, How much will Blue Panda pay to the underwriter on a per-share basis? 597,90 598,11 $1.79 \$2.10 After it pays its underwriter, how much will-Blae Panda recelve from each share of preferred stock that it issues? 588,11 5179 $2.10 52.31 $97.90 \begin{tabular}{|l|} \hline 6.13% \\ \hline 5.82% \\ \hline 5.52% \\ 4.90% \\ \hline \end{tabular} Bused on this information, Blue Panda's cost of preferred stock is